Solving 2nd order differential equation

Click For Summary
The discussion focuses on solving a second-order differential equation, where the homogeneous solution is derived from the characteristic equation yielding roots of e^{ix} and e^{-ix}. The corresponding solution is identified as B sin(x) + A cos(x). A particular solution is proposed as C, which is found to be -1, leading to the inhomogeneous solution A cos(x) + B sin(x) - 1. Participants emphasize the importance of verifying that the solution satisfies both the original differential equation and any initial conditions. This approach ensures the correctness of the derived solution.
Pushoam
Messages
961
Reaction score
53

Homework Statement


upload_2017-12-27_23-44-42.png


Homework Equations

The Attempt at a Solution



For the homogeneous equation, I have got the the root of the characteristic equation as ## e^{ix}, e^{-ix} ## .

So, the corresponding solution is ## B \sin{ x} + A \cos{ x} ## .

Then, I took the particular solution as C.

Putting C in the differential equation , I get C = -1.

So, the inhomogeneous solution is ## A \cos{ x} + B \sin{ x} – 1 ## , i.e. option(e).

Is this correct?
 

Attachments

  • upload_2017-12-27_23-44-42.png
    upload_2017-12-27_23-44-42.png
    7 KB · Views: 1,025
  • Like
Likes scottdave
Physics news on Phys.org
Looks great. You can plug it into the original differential equation to make sure that your solution satisfies it.
 
  • Like
Likes Pushoam
Pushoam said:
So, the inhomogeneous solution is ## A \cos{ x} + B \sin{ x} – 1 ## , i.e. option(e).
Is this correct?
You shouldn't need to ask this. Along the lines of @scottdave's advice, whenever you get a solution to a diff. equation, you should get in the habit of checking. That way you'll know whether your answer is correct. For an initial value problem, check that 1) your solution satisfies the initial condition, and 2) your solution satisifies the differential equation.
 
  • Like
Likes Pushoam
Question: A clock's minute hand has length 4 and its hour hand has length 3. What is the distance between the tips at the moment when it is increasing most rapidly?(Putnam Exam Question) Answer: Making assumption that both the hands moves at constant angular velocities, the answer is ## \sqrt{7} .## But don't you think this assumption is somewhat doubtful and wrong?

Similar threads

  • · Replies 3 ·
Replies
3
Views
2K
Replies
3
Views
2K
Replies
1
Views
1K
Replies
2
Views
1K
Replies
7
Views
2K
Replies
2
Views
2K
  • · Replies 5 ·
Replies
5
Views
2K
Replies
10
Views
2K
Replies
8
Views
1K